Emergency Medicine CAQ HEENT

¡Supera tus tareas y exámenes ahora con Quizwiz!

Question: Why are children younger than 4 years more prone to developing a retropharyngeal abscess?

Answer: They have large retropharyngeal lymph nodes that are prone to infection, which involute after age fou

Question: What is the classic triad in Meniere's disease?

Answer: Vertigo, tinnitus and hearing loss.

Question: What are the serious sequelae associated with orbital cellulitis?

Answer: Vision loss, cavernous sinus thrombosis and meningitis.

Question: How does one safely remove eyelid oil?

Answer: With eyes closed, use a cotton swab dipped in a dilute water and baby shampoo solution to rub the eyelashes and lid margins.

Question: What is the most likely lesion in an individual with a yellow mass on the upper eyelid with a history of hyperlipidemia?

Answer: Xanthelasma.

Question: What condition is associated with clear, fluid-filled tympanic membrane vesicles and facial nerve paralysis?

Answer: Zoster oticus.

Question: What is the normal width of the retropharyngeal space on lateral neck X-ray?

Answer: 7 mm (at C2) in both children and adults.

Question: What is the normal ocular pH?

Answer: A pH of 7.0 - 7.2 is normal for the eye.

Question: What are two complications of a nasal septal hematoma?

Answer: Abscess and saddle nose deformity.

Question: What environmental disorder is acetazolamide used to treat?

Answer: Acute mountain sickness.

Question: What is the correlation between etiology and bilateral conjunctivitis?

Answer: Bilateral symptoms are usually due to infections or allergies, while unilateral symptoms are usually due to chemicals, trauma or toxins.

Question: What is the management of Ludwig's angina?

Answer: Broad-spectrum intravenous antibiotics and operative drainage.

Question: Why is warm water recommended when irrigating the external auditory canal?

Answer: Cold water can induce severe vertigo.

Question: What is the most important step in management for a patient with a chemical splash to the eyes?

Answer: Copious irrigation.

Question: What classic corneal finding is seen with herpes simplex keratitis?

Answer: Dendritic lesions.

Question: Which patients are more likely to develop an invasive otitis externa due to Pseudomonas?

Answer: Diabetics.

Question: What is the Ellis classification system?

Answer: Ellis classes describe degrees of dental fractures. Ellis class I is a fracture involving the enamel only, class II involves the enamel and the dentin, and III involves pulp exposure.

Question: What is the recommended treatment for benign paroxysmal positional vertigo?

Answer: Epley maneuver.

Question: What are some associated symptoms in gonococcal pharyngitis?

Answer: GU symptoms (dysuria, discharge), joint pain.

Question: What four populations is acute necrotizing ulcerative gingivitis (ANUG) most commonly seen in?

Answer: HIV patients, malnourished children, young adults who experience a great deal of stress, polysubstance users.

Question: What are the complications of otitis media?

Answer: Hearing impairment, TM perforation, labyrinthitis, facial nerve palsy, meningitis, extradural abscess, mastoiditis, and lateral venous sinus thrombosis.

Question: What are two life-threatening complications of candidial esophagitis?

Answer: Hemorrhage and perforation.

Question: Vaginal delivery is contraindicated with which active sexually transmitted disease?

Answer: Herpes vaginalis infection.

Question: What is a dependent pocket of pus seen in the anterior chamber called?

Answer: Hypopyon

Question: Which underlying medical conditions are common in patients who develop necrotizing (malignant) otitis externa?

Answer: Immunocompromised patients such as diabetic patients, patients with acquired immunodeficiency syndrome (AIDS), and those receiving chemotherapy.

Question: What is the most common cause of tympanic membrane perforation?

Answer: Infection.

Question: What other entities are cell and flare seen in?

Answer: Iritis, uveitis, and postsurgical.

Question: What environmental accident is associated with tympanic membrane perforation?

Answer: Lightning strike.

Question: What are three complications of bacterial sinusitis?

Answer: Meningitis, orbital cellulitis and sinus bone osteitis.

Question: What effect do mydriatics have on patients at risk for acute angle glaucoma?

Answer: Mydriatics can precipitate acute angle glaucoma and should be avoided in patients with narrow anterior chambers.

Question: What is the intraocular pressure found in acute conjunctivitis

Answer: Normal (10-20 mmHg).

Question: What causes a chalazion?

Answer: Obstruction of the meibomian gland causing swelling sparing the lid margin.

Question: What is the other name for exophthalmos?

Answer: Proptosis, although this term is usually reserved for non-endocrine causes of globe protrusion, while exophthalmos is specifically used for globe protrusion due to an underlying endocrinopathy.

Question: What pathogens most commonly cause otitis externa?

Answer: Pseudomonas aeruginosa, Enterobacteriaceae and Proteus species, Staph. aureus.

Question: What are the common findings found on fundoscopy in central retinal artery occlusion?

Answer: Retinal edema with a pale appearance and a cherry-red spot representing the fovea.

Question: Name two common viral causes of prenatal hearing loss?

Answer: Rubella and cytomegalovirus.

Question: What is the most commonly isolated pathogen obtained from culture at bronchoscopy in patients with bacterial tracheitis?

Answer: S. aureus.

Question: What are the most common organisms that cause orbital cellulitis?

Answer: Staphylococcus aureus and Streptococcus pneumoniae.

Question: What bacterial pathogen is implicated in hordeolum and chalazion?

Answer: Staphylococcus species.

Question: What is Seidel's sign?

Answer: Streaming of aqueous humor leaking from the anterior chamber during fluorescein examination secondary to penetration of the cornea.

Question: What are the 3 most commonly identified bacterial agents in acute otitis media?

Answer: Streptococcus pneumoniae, Haemophilus influenzae, and Moraxella catarrhalis.

Question: What is the treatment for cases of mastoiditis that are refractory to parenteral antibiotics?

Answer: Surgical irrigation and debridement with possible mastoidectomy.

Question: What diagnosis should be suspected in a patient older than 50 years of age who has an ESR greater than 50?

Answer: Temporal (Giant cell) arteritis.

Question: What cranial nerve is most commonly affected in necrotizing otitis externa?

Answer: The facial nerve.

Question: What counseling should you offer parents of children with electrical burns of the lip?

Answer: The labial artery typically bleeds two to five days after the injury. You should teach them how to hold pressure on the area should this occur.

Question: In retinal detachment with maintained visual acuity, what has happened to the macula?

Answer: The macula is likely still attached if visual acuity is maintained. Urgent surgery needed to prevent macula from detaching too.

Question: All topical ocular medications with a red-colored cap have what property?

Answer: They are all mydriatics.

Question: What is the normal intraocular pressure of the eye?

Answer: 10-20 mm Hg.

Which of the following signs has the greatest likelihood ratio for acute otitis media? Impaired mobility of the tympanic membrane Red tympanic membrane Retracted tympanic membrane Ruptured tympanic membrane

Correct Answer ( A ) Explanation: A tympanic membrane with distinctly impaired mobility has an adjusted likelihood ratio of 31. A tympanic membrane with slightly impaired mobility has an adjusted likelihood ratio of 4. A tympanic membrane with normal mobility is unlikely to be consistent with acute otitis media. A distinctly red tympanic membrane (B) has an adjusted likelihood ratio of 2. A retracted tympanic membrane (C) has an adjusted likelihood ratio of 0.6. A ruptured tympanic membrane (D) is nonspecific for acute otitis media. Another sign that suggests acute otitis media is a cloudy tympanic membrane with an adjusted likelihood ratio of 34.

72-year-old man presents with a painful red eye and visual loss worsening over the last 24 hours. He recently had cataract surgery. Examination of the eye reveals the image above. Which of the following is the most likely? Endophthalmitis Hyphema Uveitis Vitreous hemorrhage

Correct Answer ( A ) Explanation: Endophthalmitis is an infection involving the anterior, posterior and vitreous chambers of the eye. It results from trauma (blunt globe rupture, penetrating injury, foreign bodies) and also iatrogenically after ocular surgery like cataract repair. Patients complain of severe pain in the eye and visual impairment or loss. Examination of the eye reveals decreased visual acuity, injected conjunctiva, chemosis and haziness of the infected chambers. Infections are treated with both systemic and intraocular antibiotics. A hyphema (B) is blood in the anterior chamber usually caused by trauma. When the patient is in an upright position, blood will layer along the inferior aspect of the anterior chamber. As the hyphema increases in size, it elevates intraocular pressure. In some cases admission is warranted for patients with large hyphemas (>50%), decreased vision, sickle cell disease and elevated intraocular pressure. Treatment is aimed at decreasing pressure with topical (beta-blocker, alpha agonist or carbonic anhydrase inhibitors) and systemic therapy (carbonic anhydrase inhibitor, mannitol). Uveitis (C) occurs after blunt trauma in which the iris and ciliary body are inflamed causing ciliary spasm. Patients complain of significant photophobia with significant eye pain. Examination of the eye reveals perilimbal conjunctival injection (also called ciliary flush) and a small, poorly dilating pupil. Photophobia occurs with light shone on both the affected and unaffected eye. On slit lamp, cells (white and red) and flare (protein) are noted in the anterior chamber. Treatment is with a topical cycloplegic agent to minimize spasm. Vitreous hemorrhage (D) occurs as a result of injuries to the retina, uveal tract and their associated vascular structures. Common associated conditions include diabetic retinopathy, retinal vein occlusion and trauma. Patients complain of decreased visual acuity and floaters. The condition is not typically painful. Diagnosis is made with ocular ultrasound showing blood products in the posterior chamber.

29-year-old woman presents with itchy eyes for the past four days. She reports "always rubbing them" as it feels like "sandpaper in there". Inspection reveals eyelid margin erythema and swelling, but no discrete nodule, mass or cyst. You appreciate crusting at the base of the eyelashes. An everted-lid examination reveals no inflammation. Sclerae are not injected. Which of the following is the most likely diagnosis? Blepharitis Conjunctivitis Molluscum contagiosum Stye (hordeolum)

Correct Answer ( A ) Explanation: Eyelid inflammation is called blepharitis. Several types exist: seborrheic, staphylococcal, mixed and parasitic. There is also an association with dandruff and acne. Onset can be acute and self-limited, resolving in 2-4 weeks, however, most problematic cases are chronic in nature. Symptoms usually include eye burning and grittiness, tearing, irritation, erythema and crusting along eyelash roots. Seborrheic types commonly result in scales. Meibomitis is likely a better term used in a patient with blepharitis who also has an expressible milky, toothpaste-like exudate from the eyelids. Most cases of blepharitis respond well to basic lid hygiene, which includes make-up avoidance and warm wet compresses followed by mechanical removal of oil. If refractory, prescription antibiotic ointment may be necessary. Conjunctivitis (B) is inflammation of the conjunctiva, the coverings of the sclerae and ocular surfaces of the eyelids. This patient has no inflammation in these areas. Molluscum contagiosum (C) is a common viral infection which results in tiny, multiple waxy nodules with central umbilication. These nodules can present on the eyelids. A stye (D), or hordeolum, is a painful cyst or mass on the eyelid margin that may be filled with serous fluid or pus. This patient has no such mass.

55-year-old man presents with amaurosis fugax, headaches, scalp tenderness, jaw claudication, occasional ear pain, malaise, and intermittent fevers. On exam, you note tenderness of his left temple. Laboratory studies reveal an erythrocyte sedimentation rate of 85. Which of the following is the most likely diagnosis? Giant cell arteritis Multiple sclerosis Myasthenia gravis Polymyositis

Correct Answer ( A ) Explanation: Giant cell arteritis (temporal arteritis) is a systemic autoimmune disorder. Pathologically, there is a granulomatous inflammation of large and medium-sized arteries. It is most common in women >50 years of age. Involvement may occur in any organ system, but the condition is characterized by subacute inflammation of the external carotid arterial system and vertebral arteries. Patients often describe a new temporal or diffuse headache that may be associated with transient visual loss, scalp tenderness, and jaw claudication. Patients may report a history of polymyalgia rheumatica (fever, myalgias, anorexia, weight loss, and arthralgias). If the diagnosis is suspected, treatment should be immediately initiated by administering steroids (prednisone 60 mg daily). Confirmation of the diagnosis is by temporal artery biopsy. Lab findings supportive of the diagnosis is an elevated ESR (50-100). Using the Westergren method, the value for a normal ESR is 30 mm/hr for a 60-year-old man; for women, top of normal range is age plus 10 divided by 2, so 35 mm/hr is the upper range of normal for a 60-year-old woman. Myasthenia gravis (C) is characterized by unilateral or bilateral ptosis, which is usually asymmetric (not amaurosis fugax), lid fasciculations, and lid retraction. Patients with myasthenia gravis frequently have eye muscle deviations with diplopia. Systemic manifestations include involvement of the jaw and neck, inability to hold the body erect because of weakness of the spinal muscles, and limb weakness. Multiple sclerosis (B) is associated with optic neuritis which is localized inflammation of the optic nerve sheath, resulting in reduced neuronal transmission and decreased visual acuity. Generally, there is a loss of color vision and red desaturation noticed by the patients. The symptoms generally worsen during the first few days and progressively improve over several weeks Polymyositis (D) is an inflammatory disease that has a bimodal distribution (age 10 to 15 and 45 to 60 years). The cause of inflammatory myopathies is unknown, but evidence suggests a genetic predisposition (associated with certain HLA markers) combined with an environmental insult, such as viruses, thereby initiating an autoimmune process. Patients usually experience progressive, symmetric, proximal muscle weakness with fatigue, malaise, and morning stiffness.

A 15-year-old boy presents with acute onset, atraumatic unilateral eyelid swelling. Examination reveals supratemporal orbital edema and tenderness. Which of the following is the most likely diagnosis? Dacryoadenitis Dacryocystitis Exopthalmos Ptosis

Correct Answer ( A ) Explanation: Inflammation of the lacrimal glands is called dacryoadenitis. It is most commonly due to viral or bacterial infection. Unilateral, supratemporal orbital pain and swelling predominate. There may be preauricular lymphadenopathy, discharge and excessive tearing. The differential includes orbital cellulitis, chalazion, hordeolum, exophthalmos, ptosis and lacrimal tumor. Any discharge should be cultured. CT evaluation is considered if tumor is suspected. Acute cases are treated with warm compresses, NSAIDs and cephalosporins if underlying bacterial infection is suspected. Chronic cases should involve evaluation of underlying disorders such as thyroid disease and sarcoidosis. Dacryocystitis (B) is an infectious obstruction of the nasolacrimal duct. onset is usually rapid with a clinical presentation of unilateral, severe pain, redness and epiphora (overflow of tears). This condition occurs in the inframedial region of the orbit. Exopthalmos (C), commonly seen in Grave's disease, is anterior bulging of the eyeball out of the orbit (globe protrusion). Ptosis (D) is drooping or falling of the upper eyelid.

Ludwig's angina is associated with which of the following? Airway obstruction Thumbprint sign on radiography Unilateral neck swelling Uvular shift

Correct Answer ( A ) Explanation: Ludwig's angina is a bilateral cellulitis, abscess formation, or both, of the submandibular and sublingual spaces (floor of the mouth) with associated tongue elevation. It is polymicrobial in origin and occurs most commonly in men aged 20 to 60 years. The infection is rapidly spreading without lymphatic involvement and generally without abscess formation. Dental infections account for approximately 80% of Ludwig's angina cases. Patients present clinically with fever, trismus, dysphonia, and odynophagia. Acute airway obstruction with airway compromise is a potentially life-threatening complication. The thumbprint sign (B) is characteristic of the swollen epiglottis seen on a soft tissue neck radiograph in epiglottitis. The epiglottis is not involved in Ludwig's angina. Ludwig's angina is bilateral in nature. Unilateral neck swelling (C) occurs in such entities as parapharyngeal space abscesses. Uvular shift (D) is seen in peritonsillar abscess.

A patient presents after a chemical splash to the eye. What management is immediately indicated? High-volume irrigation Pupil dilation and slit lamp examination Referral to ophthalmology Topical antibiotics and ophthalmology consultation

Correct Answer ( A ) Explanation: Patients who present with chemical exposure to the eye should undergo immediate irrigation with normal saline prior to the initiation of any other management. A rapid assessment of the pH of the conjunctiva with pH paper and application of a topical anesthetic may be rapidly performed prior to initiation of irrigation but no procedure or intervention should delay irrigation. Irrigation should continue for a minimum of 30 minutes followed by a check of the pH. Irrigation should be continued until the pH normalizes. Once the pH normalizes, application of a cycloplegic, topical antibiotics and a complete ocular assessment should be performed. After immediate management, focus should be placed on identifying the substance the patient was exposed to and ophthalmologic consultation for any significant injuries. Typically, alkaline substances with a pH less than 12 and acidic substances with a pH greater than 2 are not thought to cause significant injury. This may be altered if the duration of contact was prolonged. Long-term complications include perforation, scarring and neovascularization of the cornea. Glaucoma and cataracts can also occur. Although an ophthalmology assessment (C) is necessary, it should not delay irrigation and normalization of pH as this is the most vital intervention to prevent further damage. Topical antibiotics (D) will likely be needed if there is a corneal abrasion or ulceration but should not be applied until the pH is normalized. Pupil dilation and slit lamp examination (B) is time consuming and should be deferred until irrigation is completed.

Which of the following is true regarding corneal abrasions? Antipseudomonal antibiotic drops should be given to patients with contact lenses Eye patch should be given to all patients Timolol drops are effective in reducing pain Topical anesthetic agents should be prescribed

Correct Answer ( A ) Explanation: Patients who wear contact lenses and develop corneal abrasions should be treated with topical antibiotic drops with antipseudomonal activity as infection with these organisms is more common in contact wearers. Patients with corneal abrasions present with pain, decreased visual acuity and photophobia. All patients with contact lenses should be treated with topical antibiotics covering pseudomonal strains. Eye patching (B) is not recommended as it has no benefit. Timolol (C) drops are used in the treatment of glaucoma. Topical anesthetic agents (D) are not recommended as they may, theoretically, cause worsening of abrasions.

A 37-year-old man with a history of asthma presents to the ED with the finding seen in the image above. Which of the following statements best describes this patient's diagnosis? AMicroscopic evaluation will reveal pseudohyphae BMost commonly seen in healthy middle-aged adults CThe white plaques do not scrape off with a tongue depressor DTreatment includes a 7-day course of valacyclovir

Correct Answer ( A ) Explanation: The white, flaky, curd-like plaques seen in the image are consistent with oral candidiasis (thrush). It is caused by the organism Candida albicans, a type of yeast that can be identified by visualization of pseudohyphae under the microscope. The white lesions tend to be painless but can be associated with painful ulcerations, particularly in immunocompromised adults, and chest discomfort when associated with esophageal extension. Oral candidiasis occurs mainly in (B) premature infants and immunocompromised individuals (cancer, HIV, steroid-therapy). It is often the first sign of AIDS in young adults. The white plaques do scrape off (C) in oral candidiasis. This is one way to distinguish the lesions from similar looking oral hairy leukoplakia, which do not scrape off. Treatment is with oral antifungal medication (D) such as nystatin, fluconazole, or clotrimazole.

3-year-old boy presents with bleeding from the mouth after a fall. On examination, he has a completely avulsed front tooth. His mom has the tooth in her hand. The patient is otherwise well appearing and the bleeding has stopped. What management is indicated? Do not replace tooth and arrange follow up Place tooth in water and send to dentist in the morning Replacement of the tooth into the socket Scrub tooth thoroughly and replace into socket

Correct Answer ( A ) Explanation: This boy presents with a completely avulsed primary tooth and requires urgent dental referral for further care. Avulsed teeth are a true dental emergency in adults but not in young children with primary teeth. For every minute that a permanent tooth remains out of the socket, the chance of successful reimplantation goes down by 1%. Thus rapid replacement is indicated. Primary teeth, however should not be placed back into the socket. Reimplanted primary teeth may ankylose or fuse to the bone leading to dentofacial complex abnormalities. Additionally, a reimplanted primary tooth may interfere with eruption of the permanent tooth. Either way, cosmetic deformity results. Typically, primary teeth are present from six months to six years. Patients with avulsion of a primary tooth should be urgently referred to a dentist for further care. Replacement of the tooth (C) should not be undertaken as it is likely to lead to more severe dental issues. If a permanent tooth is avulsed and cannot be reimplanted, it should be stored in Hank's solution (balanced pH cell culture medium) or milk if Hank's solution is not available. Storage in plain water (B) is not recommended. Saliva is another acceptable medium when no other options are available. Prior to reimplantation of a permanent tooth, the tooth should be gently rinsed of any contaminants. Scrubbing (D), however, is not recommended as it may lead to destruction of any of the remnant periodontal ligament fibers, which can aid in healing.

18-year-old man presents with mouth pain for two days. He complains of associated fever, malaise, and a foul metallic taste in his mouth. On examination, you note poor dentition and fetid breath. He has pseudomembrane formation with gingival ulcerations and cervical adenopathy. Which of the following is the most likely diagnosis? Acute necrotizing ulcerative gingivitis Bulimia Diphtheria Ludwig angina Oral candidiasis

Correct Answer ( A ) Explanation: This patient has acute necrotizing ulcerative gingivitis (ANUG), also referred to as trench mouth. Poor oral hygiene, tobacco use, and immunocompromised states all predispose to ANUG. Anaerobic fusobacterium and spirochetes (Borellia sp.) are the predominant bacterial organisms involved. Painful and severely edematous interdental papillae characterize ANUG. Oral ulcers with an overlying grayish pseudomembrane and a "punched out" appearance are also characteristic. The inflamed gingival tissue is friable and necrotic, and represents an acute destructive disease process of the periodontium. Patients often exhibit fever, malaise, and regional lymphadenopathy and describe a foul-smelling breath and metallic taste in their mouth. The condition is treated by warm saline irrigation of the mouth and antibiotics with oropharyngeal coverage (penicillin, clindamycin, or metronidazole). Hydrogen peroxide or chlorhexidine oral rinses are also useful. Advanced disease requires consultation with an oral surgeon for dental extraction. Bulimia (B) is characterized by binge eating with self-induced vomiting. Patients are at risk for damage to the dental enamel and dentin as a result of repeated episodes of vomiting with chronic exposure to regurgitated gastric acid contents. The lingual dental surfaces are most commonly affected. The toxin-producing bacteria Corynebacterium diphtheria (C) causes diphtheria. It may involve any mucous membrane but most commonly affects the mucosa of the upper respiratory tract. It typically produces an ulcerated pharyngeal mucosa with a white to gray pseudomembrane, classically associated with a "wet mouse" odor. Unlike ANUG, the gingiva is not involved. Ludwig's angina (D) is an extensive cellulitis of the submandibular and sublingual spaces with associated tongue elevation. It is polymicrobial. Patients generally present with fever, trismus, dysphonia, odynophagia, and drooling. Oral candidiasis (E) is associated with white, flaky, curd-like plaques covering the tongue and buccal mucosa with an erythematous base. The white plaques can be removed with a tongue depressor, unlike the plaques of hairy leukoplakia, an opportunistic infection caused by Epstein-Barr virus.

65-year-old woman complains of acute pain in the left eye and vomiting after walking into a movie theater. On examination, her cornea is cloudy. She has photophobia and a fixed pupil. She reports an allergy to sulfa. What is the most appropriate action? Administration of acetazolamide Measurement of intraocular pressure Topical mydriatic agent Ultrasound of the eye

Correct Answer ( B ) Explanation: Acute angle-closure glaucoma occurs classically when patients move into a situation with lower light requiring dilation of the pupil and then obstruction of aqueous humor outflow track. Patients experience the abrupt onset of symptoms including eye pain, headache, vomiting, and blurred vision. Patients may also describe seeing a halo around lights. On examination, the conjunctiva is injected with a cloudy appearance to the cornea. The pupil is typically mid-sized or dilated and either fixed or sluggishly reactive. One must suspect the diagnosis clinically and then measure and document the intraocular pressure. Goals of treatment include rapid reduction of intraocular pressure through both topical and intravenous agents. Topical agents include beta-blockers (e.g. timolol), miotic agents (pilocarpine), and steroids. In addition, acetazolamide (carbonic anhydrase inhibitor) is commonly used to decrease aqueous humor production. Mannitol (an osmotic diuretic) can be used as an intravenous agent for severe cases. Definitive correction requires surgery.

26-year-old man presents with two days of left ear pain. He notes that the symptoms started with an itchy ear that progressed to pain, discharge, and mild hearing loss. On examination there is tenderness with manipulation of the auricle, edema, erythema, and narrowing of the tympanic canal. Which of the following is useful in treating this condition? Acetic acid otic washes Antihistamines Oral amoxicillin Tympanostomy tubes

Correct Answer ( A ) Explanation: This patient has otitis externa, also called "swimmer's ear." This infection is characterized by pruritus, pain, and tenderness of the external canal. Trauma and excessive moisture are common precipitants of this condition. Patients may present with otorrhea, otalgia, and tenderness with manipulation of the pinna (especially the antitragus). With progression, pain may be present with mastication or any movement of the periauricular skin. When chronic, narrowing of the tympanic canal and hearing impairment may occur. There are many different treatment modalities for otitis externa. Though topical antibiotics are typically administered, acetic acid drops are effective and may be used, particularly in patients who cannot afford prescription medications. Antihistamines (B) are not used in the treatment of otitis externa. They are recommended in another otic condition, Ménière's disease. Oral agents such as amoxicillin (C) are reserved for febrile patients and those with suspected concurrent otitis media. While ear wicks can assist with administration of topical medications, tympanostomy tubes (D) do not have a role in otitis externa.

A 55-year-old woman presents to the ED with acute loss of vision in her right eye. She describes cloudiness in her visual field associated with mild pain and a headache. On examination, her visual acuity is markedly decreased in the affected eye, and she has a mid-dilated and sluggishly reactive pupil. What is the most likely diagnosis? Acute angle closure glaucoma Central retinal artery occlusion Central retinal vein occlusion Optic neuritis

Correct Answer ( A ) Explanation: This patient hasacute angle closure glaucoma, which is associated with apainful vision loss,headache,nausea, and vomiting. It is caused by an outflow obstruction of aqueous humor from the anterior chamber to the canal of Schlemm. As a result, intraocular pressure (IOP) risesand thepupil becomes mid-dilated with sluggishness or nonreactivity to light. There is oftenciliary flush,and thecornea appears steamy or hazy. Patients often describe blurry vision withhalos around lights. Central retinal artery occlusion (B) is associated with a sudden painless monocular vision loss, whereas central retinal vein occlusion (C) is associated with varying degrees of severity of monocular vision loss. Central retinal vein occlusion usually has a slower onset than arterial occlusion. Key point is that vision loss of acute angle glaucoma is painful. Optic neuritis (D) can also cause acute vision loss with pain, but the latter is typically associated with extraocular movements. In addition, an afferent pupillary defect is usually present.

34-year-old woman presents to the ED with severe right eye pain. The pain began after she entered a dark movie theatre. On physical exam, you note a mid-dilated pupil (4 mm) and corneal edema. Which of the following is the correct combination of medications in the treatment for this process? Acetazolamide IV, topical apraclonidine, topical timolol, topical pilocarpine Mannitol IV, topical pilocarpine, topical timolol, prednisolone IV Topical prednisolone, mannitol IV, topical timolol, topical pilocarpine Topical timolol, topical pilocarpine, topical apraclonidine, topical prednisolone

Correct Answer ( A ) Explanation: This patient is exhibiting signs and symptoms of acute angle closure glaucoma. Acute angle closure glaucoma is defined by increased intraocular pressure caused by obstruction of aqueous humor drainage via the canal of Schlemm. The anterior chamber "angle" refers to the angle formed by the cornea and the iris. The ciliary body produces aqueous humor and is located posterior to the iris. The canal of Schlemm is located in the angle of the anterior chamber and serves to drain the aqueous humor in the anterior chamber. Contact between the iris and cornea due to anterior movement of the iris obstructs the canal of Schlemm. This happens when either there is increased pressure in the posterior chamber or pupillary dilation occurs causing narrowing of the anterior chamber and thus obstruction of the canal. Signs and symptoms of acute angle closure glaucoma include ocular pain, nausea and vomiting, unilateral blurring of vision, photopsia or colored halos around lights, elevated intraocular pressure (often 30 mm Hg or higher), conjunctival injection, a cloudy or steamy cornea due to corneal edema, and a mid-dilated nonreactive pupil. Management of acute angle closure glaucoma includes several treatment modalities. First, you need to reduce the production of aqueous humor using oral or intravenous acetazolamide, a topical beta blocker such as timolol, and a topical alpha agonist such as apraclonidine. Additionally, administration of topical pilocarpine is used to help facilitate outflow of aqueous humor through miosis. These are typically given one hour after the start of treatment and require two doses 15 minutes apart. Adjuncts, including hyperosmotic agents like intravenous mannitol, may be needed to further reduce the intraocular pressure two hours after the initiation of treatment if not adequately controlled. Ophthalmology should be consulted as soon as this diagnosis is suspected for definitive laser iridotomy Topical prednisolone, mannitol IV, topical timolol, topical pilocarpine (C) delays the reduction of aqueous humor via topical timolol; however, mannitol is reserved for use if the initial measures (acetazolamide) are unsuccessful. Topical timolol, topical pilocarpine, topical apraclonidine, topical prednisolone (D) contains a large group of correct topical medications; however, it is missing acetazolamide, which is first-line treatment in acute angle closure glaucoma. Mannitol IV, topical pilocarpine, topical timolol, prednisolone IV (B) relies on mannitol, which is an osmotic agent, over acetazolamide which specifically targets aqueous humor production. As above, mannitol should not be given until acetazolamide has failed. Furthermore, prednisolone given IV is not useful in the treatment of acute angle closure glaucoma.

A previously healthy 18-year-old woman presents with sore throat and pain with swallowing. Her vital signs are T 102.7°F, HR 124, BP 123/76, RR 22, and oxygen saturation 97%. On examination she has trismus, pain with neck extension, and difficulty swallowing her saliva. Her oropharyngeal examination is unremarkable. Which of the following is the most appropriate next step in management? CT scan of the neck with contrast and ENT consultation Ibuprofen, dexamethasone, and a Rapid strep test Oral antibiotics and ENT follow up Peritonsillar needle aspiration

Correct Answer ( A ) Explanation: This patient is suffering from a retropharyngeal abscess and will need advanced imaging (CT scan of the neck with IV contrast) to further delineate the extent of the disorder along with emergent ENT consultation for possible operative intervention. Historically, this was a disease of children under 6 years of age but adults are increasingly affected. A number of infectious processes including nasopharyngitis, otitis media, peritonsillar abscess, dental infections as well as iatrogenic procedures including endoscopy and dental instrumentation have been associated with retropharyngeal abscess formation. The infection is most commonly polymicrobial with both aerobes and anaerobes requiring broad antibiotic coverage. Patients typically present with sore throat, odynophagia, dysphagia, drooling, muffled voice, neck stiffness, fever and trismus. In severe cases, the patient may hold the neck in extension in order to increase airway diameter by distracting the posterior pharynx from the airway. CT scan and MRI are diagnostic but in unstable patients, lateral neck X-ray can demonstrate retropharyngeal swelling supporting the diagnosis. Additionally, if the patient is unable to lie flat for advanced imaging, direct visualization with an upper airway scope can be diagnostic. Oral antibiotics (C) alone are insufficient for treatment of retropharyngeal abscess in the majority of cases. In addition, the patient should not be discharged home as she is at risk to develop a compromised airway. Ibuprofen, dexamethasone, and a Rapid strep test (B) is the standard treatment for simple pharyngitis. However, this patient has a deep space infection and requires imaging, intravenous antibiotics, ENT consultation and possible surgery. In this patient, there is no swelling of the tonsils to suggest a peritonsillar abscess (D) as the cause of the patient's symptoms.

33-year-old man presents with right eye pain, swelling and fever for two days. Physical examination reveals upper and lower lid erythema and swelling. The patient has difficulty opening his eyelid and is unable to move the right eye laterally. What management should be initiated? CT scan of the orbit and intravenous antibiotics Oral antibiotics and ophthalmology follow up Topical antibiotics and ophthalmology follow up Warm compresses

Correct Answer ( A ) Explanation: This patient presents with signs and symptoms of orbital (postseptal) cellulitis and should receive imaging looking for an abscess and intravenous antibiotics. Patients with orbital cellulitis will present with systemic symptoms, proptosis, pain and limitation of extraocular movements. In the presence of these findings, patients should have CT scan of the orbits and brain performed looking for abscess near the orbit and intracranial complications. Intravenous antibiotics should be started and the patient should be admitted to the hospital with an emergent ophthalmologic consultation. The most common bacterial organisms implicated in orbital cellulitis are Streptococcus and Staphylococcus species.

A 36-year-old woman presents to the ED complaining of decreased hearing and increased fullness to the right ear. Over the last week, she has used cotton-tipped applicators to attempt to remove cerumen from her right ear. On exam, you notice a cerumen-impacted external canal on the right. You irrigate the right ear with warm saline using an 18-gauge IV catheter and a plastic curette to remove the cerumen. During the procedure, the patient has sudden decreased hearing to the right ear. Which of the following is the most appropriate next step in management? Admit the patient to the hospital and arrange for an ENT consultation Discharge home with a course of otic antibiotics drops Have the patient keep the ear dry and arrange ENT follow-up Place a cotton ball in the ear and discharge home

Correct Answer ( C ) Explanation: The patient is complaining of symptoms consistent with iatrogenic tympanic membrane perforation that occurred during disimpaction and water irrigation. Tympanic membrane perforations (TMPs) can result from a complication of infection (acute otitis media, myringitis); blast injury (explosion, slap, lightning); barometric pressure changes (flying in airplane, scuba diving); and improper attempts at wax removal or ear cleaning. The pars tensa is the most common area of the TM to perforate because it is the most anterior and thinnest portion. Patients typically experience decreased or complete hearing loss, pain, and bleeding. In the setting of tympanic membrane perforation, the goal is to keep the ear dry and arrange for follow-up with an ENT. Prophylactic antibiotics are not necessary. Most heal within a few months The patient does not require admission (A) to the hospital. ENT care can be arranged for as an outpatient. Traumatic tympanic membrane perforations only require otic antibiotics (B) when the ear is contaminated, foreign material is suspected of remaining in the middle ear, or the rupture is secondary to infection, and they require ENT follow-up. The patient should receive more than a cotton ball (D) in her ear. Her management should include analgesia and ENT follow-up because complications of tympanic membrane rupture include facial nerve palsy, vertigo, and hearing loss.

A teenager presents with eye pain and redness but no pruritis. He reports this began acutely yesterday when he woke up. He describes some crusting which made it difficult for him to open his eyes. He plays football, and frequently has helmet contact, but he denies any loss of consciousness or specific orbital trauma. Interestingly, he does report most of the team has been sick since their last road trip. Examination reveals thick, boggy conjunctiva with purulent discharge. Which of the following is the most likely etiology of his condition? Allergic reaction Bacteria Chemical Virus

Correct Answer ( B ) Explanation: Conjunctivitis, a catch-all term that refers to inflammation of the conjunctiva and the small conjunctival vessels, is mostly caused by viral or bacterial etiologies. Bacterial conjunctivitis is typified by acute onset, minimal pain, minimal pruritis, purulent discharge and thick, boggy conjunctiva (chemosis). An exposure history is more common in bacterial cases. Corneal surface diseases, such as keratitis or abrasion, will likely lead to the bacterial type over other types of conjunctivitis. Allergic (A) conjunctivitis is characterized by acute or subacute onset, no pain and no exposure history. Severe pruritis is the major differentiating symptom. There is no chemical exposure given, as such, chemical (C) conjunctivitis is unlikely. Viral (D) conjunctivitis is characterized by subacute onset, minimal pain, moderate pruritis, watery discharge, photophobia and foreign-body sensation. Lymphadenopathy and prior upper respiratory infection are also common. Furthermore, a self-innoculation history, in which one eye is affected, then improves, then the other eye becomes affected, is more common in the viral type.

A previously healthy 9-year-old boy presents with a sore throat. He denies coryza and diarrhea. He has a fever of 102.3° F and tender cervical lymphadenopathy. Posterior pharynx inspection reveals white exudates. Which of the following is the most likely cause of this patient's symptoms? Candida albicans Group A beta-hemolytic streptococcus Mycoplasma pneumoniae Rhinovirus

Correct Answer ( B ) Explanation: Most cases of pharyngitis are viral. However, the most common bacterial cause, Group A beta-hemolytic streptococcus (GABHS), is the cause of up to a third of childhood and tenth of adulthood cases. Transmission is via respiratory droplets, and incubation is 2-3 days. Symptoms include temperature greater than 100.4°F, tonsillar exudates, cervical adenopathy, and lack of cough. Diagnostic tools include throat culture and rapid antigen detection. Other than supportive measures, antibiotic treatment depends on several factors which determine the likelihood of bacterial infection. Penicillin, amoxicillin and first-generation cephalosporins are the drugs of choice. For penicillin allergic patients, azithromycin or clindamycin are alternatives. Candida albicans (A) can cause pharyngitis and pharyngeal exudates, however, it is almost always associated with an immunocompromised state, such as patients on chronic corticosteroids, or those with organ transplantation, or HIV infection. This patient has no such immunosuppression. Mycoplasma pneumoniae (C) is a very rare cause of bacterial pharyngitis. In addition to sore throat, the presence of cough, coryza and diarrhea are more indicative of a viral, not bacterial, pharyngitis. Rhinovirus (D) and adenovirus are common causes of viral pharyngitis.

Which of the following tests is used to diagnose benign paroxysmal positional vertigo? Caloric testing Dix-Hallpike maneuver Head thrust test Rinne test

Correct Answer ( B ) Explanation: The Dix-Hallpike maneuver is used to reproduce vertigo in patients with positional dizziness. With the patient in a seated position, the neck is extended and turned 45 degrees to one side. The patient is then rapidly placed into a supine position with the head hanging over the edge of the bed. The patient is kept in this position for 30 seconds while evaluating for the presence of nystagmus. The test can then be performed on the other side. This maneuver will usually provoke symptoms in patients with benign paroxysmal positional vertigo. A positive test is the reproduction of vertigo and the presence of nystagmus. Caloric testing (A) is performed by infusing warm or cold water into the ear and then checking for nystagmus. Lack of response suggests a peripheral lesion as the cause of vertigo. The head thrust test (C) is used in the diagnosis of vestibular neuronitis. The head thrust test is positive when the examiner quickly turns the patient's head and they are unable to maintain visual fixation. The Rinne test (D) is performed using a tuning fork and can help to distinguish between conductive and sensorineural hearing loss.

A mother who had no prenatal care brings her 3-day-old infant to the ED secondary to eye drainage. On exam, the child has marked swelling and redness of the eyelids, severe chemosis, and purulent discharge bilaterally. Which of the following organisms is the most likely diagnosis? Chlamydia trachomatis conjunctivitis Neisseria gonorrhea conjunctivitis Staph. aureus conjunctivitis Vernal conjunctivitis

Correct Answer ( B ) Explanation: This neonate has N. gonorrhea conjunctivitis. This is an extremely aggressive form of conjunctivitis that can ulcerate and perforate an intact cornea within hours or days. It is considered an ocular emergency and is most commonly seen in newborns (ophthalmia neonatorum), typically within the first 3-5 days of life. It is also seen occasionally in sexually active adults. The discharge is characteristically purulent. Treatment includes parenteral and topical antibiotics. A Gram stain and culture should also be obtained. C. trachomatis (A) conjunctivitis is the leading cause of preventable blindness in the world. It is also commonly seen in newborns around 1-2 weeks after birth. Gram stain will be negative because Chlamydia trachomatis is an intracellular parasite. In newborns, it may also be associated with chlamydial pneumonia. Staph. aureus (C) conjunctivitis is also associated with mucopurulent discharge, but it is not typically seen in the neonatal period. White ulcers may be seen at the limbus (marginal ulcers) as a result of an allergic reaction to the staphylococcal toxin. Vernal conjunctivitis (D) is a type of allergic conjunctivitis that most commonly affects male children. It is characterized by intense itching, burning photophobia, chemosis, bilateral lid edema, cobblestone papillae under the upper lid, and a stringy mucoid discharge. A potential complication is a "shield ulcer" produced from the irritated papillae and discharge; if the ulcer becomes infected or scarred, loss of vision may occur.

39-year-old woman presents with pain and swelling around the eye as seen above. Extraocular motions are intact and vision is normal. Which of the following is the most appropriate management? CT scan of the face Oral and topical ocular antibiotics Topical antibiotics Warm compresses and ophthalmology follow up

Correct Answer ( B ) Explanation: This patient presents with dacrocystitis and should be treated with topical ocular and oral anti-staphylococcal antibiotics. Dacrocystitis is an acute infection of the lacrimal sac secondary to lacrimal duct obstruction. It is typically caused by Staphylococcus aureus. Symptoms include swelling, redness, pain and tenderness to palpation over the lacrimal sac. In addition to topical and oral antibiotics, patients should be treated with warm compresses and gentle massage of the area. Complications of improperly treated dacrocystitis include periorbital and orbital cellulitis. CT scan of the face (A) is unnecessary unless a deeper infection is suspected. Topical antibiotics alone (C) are inadequate for treatment. Warm compresses (D) can aid in treatment but are inadequate for resolution without antibiotics.

16-year-old girl presents complaining of pain behind her left ear. She thought the pain was due to an ear infection and took three of her boyfriend's leftover antibiotic tablets without seeing her primary care provider. Her ear pain improved for a couple of days, but now she is complaining of fever and discharge from the external auditory canal. Her vitals are T 38.4°C, BP 120/80 mm Hg, HR 108 bpm, and RR 18/min. On physical examination, she has postauricular tenderness, swelling, and erythema. You note purulent otorrhea through a perforated tympanic membrane. Which of the following is the most appropriate first step in management? Administer topical antibiotic suspension containing ciprofloxacin and place an ear wick Consult an ENT physician CT scan and intravenous antibiotics Restart oral antibiotic therapy and treat for 10 days Saline irrigation and suctioning of the external auditory canal

Correct Answer ( C ) Explanation: This patient has acute mastoiditis, an infection of the mastoid air cells that usually results from extension of untreated or inadequately (as in this case) treated otitis media. Patients typically present with fever, chills, postauricular (mastoid) erythema and tenderness, and discharge from the external auditory canal. It is critical to identify and treat mastoiditis because untreated mastoiditis can progress to meningitis, encephalitis, sinus thrombosis, brain abscess, facial nerve palsy, and sepsis. A contrast-enhanced CT scan of the head with inclusion of the petrous portion of the temporal bones should be obtained to confirm the diagnosis (primarily through visualization of fluid in the mastoid air cells) and identify the presence of any complications. Patients with acute mastoiditis should be started on intravenous antibiotics, and most require admission. However, chronic mastoiditis can be treated on an outpatient basis with oral antibiotics and ENT follow-up. Placement of an ear wick and topical antibiotic therapy (A) with a fluoroquinolone is appropriate treatment for otitis externa not mastoiditis. An ENT physician (B) should be consulted, but this is not the first step in the management process. Patients should first have the diagnosis confirmed by CT scan followed by treatment with intravenous antibiotics. Oral antibiotics (D) are adequate for the treatment of chronic, not acute, mastoiditis. Saline irrigation and suctioning (E) is used in patients with otitis externa to allow for a better evaluation of the external auditory canal.

20-year-old woman presents with an acute onset of dizziness. The patient describes the sensation that the room is spinning when she turns her head to the left and it is accompanied by nausea and vomiting. The symptoms resolve with turning her head away from that side. Examination reveals left-sided nystagmus elicited by movement and no other neurologic findings. What management is indicated? Brain MRI Epley maneuver Non-contrast head CT Oral steroids

Correct Answer ( B ) Explanation: This patient presents with peripheral vertigo most consistent with benign paroxysmal peripheral vertigo (BPPV) and should be treated with an Epley maneuver. Vertigo is defined as the sensation of disorientation in space combined with a sensation of motion. Patients typically describe a room-spinning sensation or the feeling of sea sickness. Vertigo can be divided into two types: central and peripheral. Central vertigo are those disorders arising from the central nervous system and include ischemic stroke, vertebrobasilar insufficiency and infectious causes (meningitis, mastoiditis, syphilis). Central vertigo is characterized by longer duration of symptoms, minimal change with position, gradual onset and multidirectional nystagmus. Peripheral vertigo includes BPPV, Menieres disease, Labyrinthitis and vestibular neuritis. Peripheral vertigo may have intermittent symptoms (BPPV) or continuous symptoms but should not be associated with other neurologic deficits or changes and should have unidirectional nystagmus. The symptoms in BPPV are elicited by specific movements of the head and relieved by returning the head to a neutral position. The symptoms should be acute in onset and of a short duration. In BPPV, the symptoms are caused by the presence of an otolith in one of the semicircular canals. Although pharmacologic intervention may be necessary in the acute setting with meclizine or benzodiazepines, the best treatment for BPPV is the Epley maneuver. The Epley maneuver is a series of positions that the clinician takes the patient through that leads to expulsion of the otolith from the semicircular canal and relief of symptoms Imaging with a non-contrast head CT (C) is not indicated in peripheral vertigo of any cause as the patients pathology is in the inner ear and not the brain. If a central cause is suspected, MRI of the brain (A) is the best test for diagnosis as the causative lesion will likely be in the posterior fossa, which is not seen well on CT scan. Steroid treatment (D) is the indicated management for vestibular neuritis but does not play a role in the treatment of BPPV.

10-year-old boy presents with a two day history of sore throat, fever and headache. He denies cough, significant rhinnorhea or head congestion. Physical exam is remarkable for enlarged, erythematous tonsils with a pharyngeal whitish exudate. He has marked lymphadenopathy over his anterior cervical lymphoid chain. What is the most likely diagnosis? Mononucleosis Sinusitis Strep throat Tonsillolithiasis

Correct Answer ( C ) Explanation: "Strep throat" caused by Group A Beta-hemolytic streptococcus (Streptococcus pyogenes or GAS) is a common etiology of acute pharyngitis especially in children ages 5 to 15. It is characterized by inflammation of the pharynx or tonsils (tonsillar exudates) associated with symptoms of fever, malaise and sore throat, as well as the absence of other URI symptoms such as nasal congestion and cough. Cervical lymphadenopathy is often found on exam, as is a whitish exudate over the pharynx and tonsils. A rapid streptococcal antigen test is recommended in order to determine if treatment with antibiotics is warranted, as other conditions which do not require antibiotic treatment may mimic streptococcal pharyngitis. A throat culture to rule-out GAS infection is recommended in children if rapid antigen testing is negative (90% sensitivity), in order to limit transmission and prevent complications such as rheumatic fever. Other complications of strep throat may include acute glomerulonephritis, peritonsillar abscess, bacteremia, sinusitis and pneumonia. Penicillin-based antibiotics (benzathine penicillin IM or oral penicillin VK) are the treatment of choices. For penicillin allergic patients, azithromycin is an alternative. Mononucleosis (A) is an infection caused by Epstein-Barr virus and is most common in 15 to 24 year-olds. Patients usually experience a several day prodrome of fever, chills, malaise and anorexia followed by the onset of throat pain, fever and lymphadenopathy. The diagnosis is supported by heterophile (Monospot) antibody testing. Treatment is supportive and the illness is generally self-limited, though some patients may experience residual symptoms of malaise and fatigue for months following initial diagnosis. Sinusitis (B) refers to inflammation of the mucous membranes lining the paranasal sinuses. Patients present complaining of cough, facial tightness, headache, reduced ability to smell and nasal congestion with purulent nasal and postnasal discharge lasting 7-10 days. Sinusitis is usually caused by a virus, leading to decreased clearance of secretions within the mucosa and entrapment of bacteria which may lead to secondary bacterial infection. The three most common bacterial agents involved in sinusitis are Haemophilus influenzae, Streptococcus pneumoniae and Moraxella catarrhalis. Treatment of viral sinusitis is supportive; antibiotics are indicated only if the clinical course strongly suggests bacterial secondary infection. Tonsillolithiasis (D) is a condition caused by chronic inflammation of the tonsils in which small concretions develop within one or both of the tonsils. Symptoms of this condition may include halitosis, foreign body sensation, dysphagia and odynophagia, otalgia, and neck pain, though many patients are asymptomatic. Definitive treatment is surgical removal, though not required if symptoms are not bothersome.

A 27-year-old healthy man presents with facial pain and low grade fever. For the last two days he has felt congested and noticed green drainage from his nose. Which of the following is the most appropriate management? Antihistamines CT scan of the sinuses Ibuprofen Systemic antibiotics

Correct Answer ( C ) Explanation: Acute sinusitis is defined as inflammation of the nasal cavity and paranasal sinuses lasting less than four weeks. The most common etiology of sinusitis is a viral infection with acute bacterial sinusitis responsible for only 0.5 to 2 percent of episodes. The most predictive signs of acute sinusitis are purulent rhinorrhea, nasal congestion and facial pain. Viral sinusitis and bacterial sinusitis are indistinguishable clinically and expert consensus recommends considering a bacterial cause once symptoms last more than 7 to 10 days. Treatment of sinusitis is aimed at symptomatic control. Once symptoms persist for more than 7 to 10 days, a bacterial etiology is possible and antibiotics should be considered. The use of topical glucocorticoids may confer some benefit although data are mixed in trials. Oral decongestant therapy also does not have significant evidence in support of their generalized use. In cases where eustachian tube dysfunction contributes to the development of sinusitis, oral decongestants appear to have benefits. The use of non-steroidal anti-inflammatory medication like ibuprofen does provide pain relief and should be prescribed to the patient. Antihistamines (A) are often prescribed to patients with sinusitis in an attempt to dry up nasal secretions. However, this benefit has not been demonstrated in trials. A CT scan of the sinuses (B) is not necessary unless there is concern for a complication of sinusitis. Acute sinusitis is a clinical diagnosis and treatment is initiated based on the clinical examination. Systemic antibiotics (D) are not indicated in the management of acute sinusitis until the symptoms persist beyond 7 to 10 days. However, studies suggest that bacterial sinusitis resolves without antibiotics in up to 70% of cases.

6-year-old boy presents to the ED with progressive difficulty breathing. His symptoms began 3 days ago as a simple URI but suddenly worsened with high fever and productive cough. On exam, you note inspiratory and expiratory stridor, a raspy voice, and a toxic-appearing child. The lateral and A-P neck radiographs are normal. Which of the following is the most appropriate next step in management? CT scan of the neck with IV contrast Intravenous dexamethasone Intubation and bronchoscopy in the operating room Nebulized racemic epinephrine

Correct Answer ( C ) Explanation: Bacterial tracheitis often occurs as a secondary infection after a viral URI. Patients will first appear to have a croup-like presentation, which suddenly worsens with high fever, stridor, and cough. Patients will have a toxic appearance. Because upper airway obstruction from thick mucopurulent secretions of the trachea occurs, bacterial tracheitis is also known as membranous laryngotracheobronchitis, or bacterial croup. Management is similar to epiglottitis, with patients undergoing intubation and bronchoscopy in the operating room. Establishing a definitive airway should not be delayed by obtaining a CT scan (A). Also, it is dangerous to lay these patients flat because this can precipitate airway compromise. Intravenous dexamethasone (B) has not been shown to improve outcomes. Antibiotics should be administered and bronchoscopy performed, which may be therapeutic by removing the purulent pseudomembrane that forms in bacterial tracheitis. Nebulized racemic epinephrine (D) is a treatment for viral croup, not bacterial tracheitis.

A woman suffers from an acute attack of vertigo, nausea, and vomiting. You suspect viral labyrinthitis. Which of the following medications is the best choice in treating the vertigo? Acyclovir Phentermine Prednisolone Prochlorperazine

Correct Answer ( C ) Explanation: Commonly a complication of otitis media, upper respiratory infection or meningitis, labyrinthitis is an abnormality of the inner ear vestibular system, which consists of three semicircular canals, vestibule, utricle and sacule. When the vestibular system is damaged, injured or inflamed, balance deficits and vertigo arise. However, since an anatomic connection between the vestibular system and cochlea exist, tinnitus and hearing impairment also occur. The classic clinical picture of labyrinthitis is acute, profound, incapacitating vertigo, with nausea, vomiting and nystagmus. Tinnitus and hearing impairment usually accompany the onset of vertigo. These symptoms resemble those of Meniere's disease, however in Meniere's disease, symptoms are intermittent and possibly less severe. Furthermore, the pathology is different between the two conditions. Labyrinthitis is mainly due to inflammation, while an excess of inner ear fluid causes Meniere's disease. Treatment of suspected viral labyrinthitis includes bed rest, hydration, antiemetics and vestibular depressants. Treatment with corticosteroids during the acute period of vertigo has been shown to improve the recovery of peripheral vestibular function in patients with acute labyrinthitis. Benzodiazepines are also effective. Currently, the role of antiviral therapy, like acyclovir (A), in the treatment of labyrinthitis is not established. Studies to date do not show an improvement over placebo. Phentermine (B) is used as an appetite suppressant and plays no role in the treatment of labyrinthitis. Prochlorperazine (D), an antiemetic, is commonly used in the treatment of acute viral labyrinthitis. Its dopamine-antagonism and anticholinergic effects help relieve nausea and vomiting, but do not diminish the distressing sensation of vertigo.

7-year-old girl with no past medical history presents with a cough and runny nose. She is well appearing with fever of 38°C but otherwise normal vital signs. Exam reveals clear lungs, nasal congestion and a red, bulging tympanic membrane. What treatment is indicated? Ceftriaxone intramuscular High-dose amoxicillin with clavulinic acid Observation and close follow up without initial antibiotics Polymixin B/neomycin/hydrocortisone drop

Correct Answer ( C ) Explanation: This patient presents with an upper respiratory infection and signs of an otitis media (OM). OM is the most common diagnosis made by US physicians in patients < 15 years of age but is most common in children < 36 months of age. Diagnosis rests on the presence of a middle ear effusion evidenced by absent or limited mobility of the tympanic membrane (TM) or the presence of a bulging TM or an air fluid level behind the TM. Additionally, the middle ear should have inflammation presenting as erythema or pain. It is important to note that the TM can become erythematous if a child is crying. Otitis media may be caused by a number of bacteria but Streptococcus pneumoniae is the most common bacterial pathogen. Other common causative organisms are Haemophilus influenzae and Moraxella catarrhalis. In the past, all OM was treated immediately with antibiotics. However, more recent recommendations recognize that many patients with OM have viral causes and do not require antibiotics. Up to 80% of cases of acute OM resolve spontaneously. The American Academy of Pediatricians currently endorses either antibiotic therapy or observation for select populations (see table below). Severe acute OM is defined as moderate to severe otalgia, otalgia for at least 48 hours or temperature > 39oC (102.2°F). If the decision is made to start antibiotics, first-line therapy should be with high-dose amoxicillin 80-90 mg/kg/day split into two doses. Ceftriaxone intramuscular (A) and amoxicillin/clavulanic acid (B) are indicated for treatment of acute OM after failure of first-line therapy. Polymyxin B/neomycin/hydrocortisone drops (D) are an appropriate treatment for otitis externa (swelling and redness of the external canal) but does not treat otitis media.

A patient presents with epistaxis following blunt trauma to the nose. Which of the following is most suggestive of a nasal septal hematoma? Bright red discoloration Complete blockage of the nasal cavity No effect on the size of the soft tissue edema with oxymetazoline Presence of nasal bone fracture

Correct Answer ( C ) Explanation: In any situation of nasal trauma, the examiner must inspect the inside of the nares for a septal hematoma. A septal hematoma looks like a large purple mass extending from the septum. In the setting of blunt trauma, submucosal blood vessels are torn and blood accumulates between the perichondrium and the cartilage of the septum leading to the hematoma. Within days, the hematoma can become infected and drainage is indicated. Additionally, the pressure of the hematoma on the septum may lead to necrosis and septal perforation. On physical examination, it is often difficult to distinguish between soft tissue edema as a result of the blunt trauma or an actual hematoma. A septal hematoma should not change size with the administration of oxymetazoline. Typically a septal hematoma does not have a bright red discoloration (A) but is more purple in color due to the accumulation of blood under the perichondrium. Both regular soft tissue swelling as well as a hematoma may cause complete blockage of the nasal cavity (B) and so this cannot be used as a discriminatory criterion. While septal hematomas are often associated with the presence of a nasal bone fracture (D), it is common to have significant soft tissue swelling without the development of a hematoma.

3-year-old boy presents with atraumatic eye pain. Examination reveals a bulging, erythematous, edematous left eye with decreased extraocular movement and surrounding eyelid edema. Which of the following is the most appropriate initial management step? Consult infectious disease service Consult ophthalmology Intravenous antibiotics Oral antibiotics

Correct Answer ( C ) Explanation: Inflammation behind the orbital septum, the fibrous portions of the eyelids which form the anterior boundary of the orbit, is called orbital cellulitis. Common causes are sinusitis, upper respiratory infections, eye trauma and sepsis. Symptoms include eye pain and limitation with orbital movement, erythema and edema, bulging of the eye and potentially a loss of vision (therefore, it is an ophthalmologic emergency). If untreated, up to 20% of patients will become blind. Inpatient treatment is recommended, and includes hospital admission, labs, cultures, intravenous antibiotics and frequent monitoring with neurologic and vision checks. If improvement is seen in 48 hours, discharge home on oral antibiotics. If no improvement in 48 hours, consider the presence of an abscess and consult surgery and an infectious disease specialist.

6-year-old girl returns from summer swimming camp with otalgia and difficulty hearing. Otoscopic examination reveals otorrhea and auditory canal erythema. The tympanic membrane is normal. Palpation of the auricle and surrounding bone is nontender, however, lateral pulling of the auricle reproduces significant pain. Which of the following is the most likely diagnosis? Auricular chondritis Mastoiditis Otitis externa Otitis media

Correct Answer ( C ) Explanation: Otitis externa is defined as inflammation of the external auditory canal. It is frequently due to bacterial pathogens, such as Staphylococci, Proteus and Pseudomonas, however, some cases are the result of infection with fungi like Candida and Aspergillus. Risk factors include cerumen impaction, instrumentation, high humidity environments and water immersion (i.e., common in swimmers). Symptoms include otalgia, otorrhea, pruritis and decreased hearing. Pain with traction on the tragus or auricle (pinna) is a key sign, and is usually accompanied by erythema, edema and otorrhea. Treatment includes avoiding water, debridement, topical corticosteroids and antibiotics. Left untreated, otitis externa can become invasive, leading to osteomyelitis, cranial nerve palsies and death, or can spread to the auricle, causing chondritis.

36-year-old Guatemalan male migrant farm worker complains of a left eye mass that is seen in the image above. Which of the following is the most likely diagnosis? Cataract Pinguecula Pterygium Xanthelasma

Correct Answer ( C ) Explanation: Pterygia are fibrovascular proliferations thought to be triggered by ultraviolet light exposure. They occur more commonly in young males from hot, dry climates. Pterygia are characterized by their shape which is widest at the bulbar conjunctiva and narrowest at the cornea. Visual impairment may occur in large pterygia, but most are asymptomatic. Surgical removal is necessary once vision is impaired. Pinguecula (B) is a raised yellowish white mass on the bulbar conjunctiva similar to pterygia, but they do not extend to the cornea. Xanthelasma (D) are also yellow masses around the eye, but they occur from elevated plasma lipid level and are seen on the canthus of the eyelid. Cataracts (A) appear as an opacity in the lens, typically in the elderly.

72-year-old man complains of painless, decreased vision in his left eye associated with flashing lights and floaters. Visual acuity in the left eye is 20/200 and in the right eye is 20/30. Which of the following is the most likely diagnosis? AAcute angle-closure glaucoma BOptic neuritis CRetinal detachment DTemporal arteritis

Correct Answer ( C ) Explanation: Retinal detachment involves a separation of the inner neuronal retina from the outer retinal pigment epithelial layer, usually as the result of a tear in the retina from vitreous traction. Early symptoms include flashing lights, the presence of "spider webs" or "coal dust" and floaters in the visual field. As the retina detaches, the patient experiences painless vision loss often described as a curtain that is gradually lowering or raising in front of the affected eye. Direct fundoscopy may reveal a dull, gray detached retina. Management involves immediate ophthalmology consultation for possible surgery or laser therapy. Acute angle-closure glaucoma (A) is due to a rise in intraocular pressure and is associated with painful monocular vision loss. It is associated with a fix mid-dilated pupil, a hazy cornea, and perilimbic injection. Optic neuritis (B) is an inflammatory process of the optic nerve. It is characterized by loss of central vision with preservation of peripheral vision. Patients often describe pain with eye movement and reduced color vision. Temporal arteritis (D) is a vasculitis that causes monocular vision loss and is associated with headache and polymyalgia rheumatica. It is more common in individuals older than 50 years.

4-year-old boy presents with throat pain. Mom indicates that the pain started four days ago and has progressively worsened. The patient is refusing to eat or drink. His temperature is 38.3°C, HR 143 bpm, and oxygen saturation is 96% on room air. On exam, he has significant pain with extension of his neck. You note cervical lymphadenopathy, a muffled voice, and some drooling. Which of the following statements is correct regarding this patient's condition? A lateral soft tissue neck radiograph can be diagnostic when taken during expiration and with the neck in flexion Poststreptococcal glomerulonephritis is a complication The retropharyngeal space at C2 is wider than 7 mm The uvula is displaced and trismus is present

Correct Answer ( C ) Explanation: Retropharyngeal abscess usually presents with fever, difficulty in swallowing, excessive drooling, sore throat, changes in voice (muffled), or neck stiffness. The child often refuses to eat. Limitation of neck movement on exam, particularly hyperextension, is a common finding. The abscess usually follows an upper respiratory infection, pharyngitis, otitis media, or a wound infection following a penetrating injury into the posterior pharynx (popsicle stick). The diagnosis is suggested by the presence of thickened prevertebral space at the C2-C4 level on a soft tissue plain-film neck radiograph. A pathologic process is suggested if the retropharyngeal space on lateral neck films (measured from the anteroinferior aspect of the second vertebral body to the posterior pharyngeal wall) is wider than 7 mm in both children and adults. If the patient is stable, an IV contrast-enhanced CT scan of the neck can be obtained to define the extent of the infection. The soft tissue neck radiograph should be taken while the patient holds his or her head in the sniffing position (slight extension of the neck) and during inspiration not during expiration in flexion (A). This helps reduce false-positive findings because the prevertebral tissue may appear falsely enlarged during neck flexion or crying (expiration). Needle aspiration should be performed only in patients with impending obstruction or failed antibiotic therapy. This should be performed in the operating room. The needle is directed into the prevertebral soft tissue. In patients with a peritonsillar abscess who are undergoing needle aspiration, the needle should be directed at the superior lateral aspect of the tonsil. It is important to be cautious of the carotid artery during this procedure. Poststreptococcal glomerulonephritis (B) is a complication of group A strep pharyngitis. It is not typically associated with a retropharyngeal abscess. Trismus and uvular displacement (D) typically do not occur together with a retropharyngeal abscess but may be seen in patients with a peritonsillar abscess

As you are irrigating the external canal of a 20 year old male to treat cerumen impaction, the patient experiences sudden hearing loss and otalgia. You stop irrigating and attempt to visualize the tympanic membrane but cannot identify it. Which of the following is the most appropriate next step in management? Continue to irrigate with warm water Emergent otolaryngology consultation Otolaryngology referral in 1 to 2 weeks Prescribe oral antibiotics and otolaryngology referral in 1 to 2 weeks

Correct Answer ( C ) Explanation: Treating cerumen impaction involves gentle irrigation at the superior portion of the external auditory canal. This technique directs the pressure of the irrigant stream toward the wall of the canal and not the tympanic membrane. The most common complication of irrigation is traumatic tympanic perforation. When this occurs, patients usually experience sudden hearing loss, severe otalgia, or vertigo. Visualizing the tympanic membrane in this scenario is sometimes difficult. Perforation of the tympanic membrane is a clinical diagnosis. In cases of suspected perforation following irrigation follow-up with otolaryngology is necessary. n suspected tympanic membrane perforation, irrigation (A) should be discontinued to avoid damaging the ossicles. An emergent otolaryngology consultation (B) is not necessary in the setting of tympanic membrane perforation, unless there is suspected damage to the ossicles. Oral antibiotics are not indicated (D) unless foreign material is suspected of remaining in the middle ear or the perforation was due to middle ear infection.

15-year-old boy presents with decreased hearing and otorrhea. On examination of the ear, you see the image above. Which of the following is safe? Antipyrine-benzocaine drops Gentamicin drops Ofloxacin otic suspension Swimming

Correct Answer ( C ) Explanation: Tympanic membrane perforations occur most commonly from infections as pressure builds up behind the membrane causing it to rupture. Perforation may also result from trauma including direct ear trauma, explosions, severe pressure (diving) and direct injury from ear cleaning or other instrumentation. In the setting of infection, copious purulent drainage in the external canal is often present. Most perforated membranes heal spontaneously without intervention. Antibiotics may be considered in the presence of an active infection. Typically the suspension form is used rather than solution due to increased viscosity and less likelihood of entering the inner ear. This may be given in combination with antibiotic drops although prophylactic antibiotics for an isolated perforation are not required. Rarely, ENT physicians may place a patch over the eardrum if healing is prolonged or ultimately repair the perforation surgically. In children with a perforated tympanic membrane after otitis media, systemic oral therapy is preferred over topical agents. The quinolones (e.g. ciprofloxacin or ofloxacin) are not toxic and can be administered to patients with perforated tympanic membranes. Antipyrine-benzocaine drops (A) are used for pain control in patients with otitis media. However, when the tympanic membrane is perforated the benzocaine portion is ototoxic and should be avoided. Gentamicin drops (B) are a member of the aminoglycoside family of antibiotics and similarly have ototoxic effects and are contraindicated. One of the most important components of treatment of a perforation is keeping the ear canal dry. Therefore, swimming (D) should be avoided. Additionally, patients should be advised to wear a shower cap when showering to prevent water from entering the canal.

65-year-old man complains of sudden pain and vision loss in his left eye about 20 minutes after entering a movie theater. Which of the following is the most appropriate treatment for his condition? ACyclopentolate BDigital massage of the globe CHigh-dose steroid DTopical beta-blocker

Correct Answer ( D ) Explanation: Acute angle glaucoma can be triggered by dim light (such as going from bright light outside to a darkened theater), which dilates the pupil leading to occlusion of the trabecular meshwork and obstruction to aqueous humor outflow resulting in increased intraocular pressure. Treatment is aimed at rapid reduction of intraocular pressure by inhibiting production of aqueous humor and improving outflow. This can be achieved with a topical beta-blocking agent such as timolol. Additional medications that may be used include alpha-agonists (brimonidine), prostaglandin analogues (latanoprost), and pilocarpine (topical miotic). Adjunctive therapy includes carbonic anhydrase inhibitors (acetazolamide) and osmotic diuretics (mannitol). Corneal indentation is reserved for intraocular pressures >50 mm Hg or when other agents are ineffective. Cyclopentolate (A) is a mydriatic agent that can induce and worsen acute angle glaucoma. As such, it should be avoided in all patients at risk for this condition. Digital massage of the globe (B) is a technique used in patients with central retinal artery occlusion to help dislodge the embolus. High-dose steroids (C) are used in patients with vision loss due to temporal arteritis.

2-year-old boy presents with right-sided ear pain. On otoscopy, you observe the image seen above. What organism is commonly associated with this condition? Mycoplasma pneumoniae Pseudomonas aeruginosa Staphylococcus aureus Streptococcus pneumoniae

Correct Answer ( D ) Explanation: Bullous myringitis is a direct inflammation and infection of the tympanic membrane caused by a viral or bacterial agent. Vesicles or bullae filled with blood or serosanguineous fluid on an erythematous tympanic membrane are the hallmark of bullous myringitis. Bullous myringitis was previously linked to Mycoplasma pneumoniae but it appears, based on middle ear aspirate culture results, that typical acute otitis media pathogens such as Hemophilus influenzae and Streptococcus pneumoniae are more likely responsible.

73-year-old man presents with painless vision loss in the right eye. Which of the following on funduscopic examination is most characteristic of central retinal vein occlusion? Cherry-red fovea Pale retina Papilledema Retinal hemorrhages

Correct Answer ( D ) Explanation: Central retinal vein occlusion occurs as a result of thrombus formation in the retinal vein. This can occur in the setting of multiple causes including mechanical compression, sluggish circulation, vasculitis, and hypercoagulability. As the central retinal vein is occluded, there is increased resistance to venous flow in the retinal venous system. The impaired bloodflow can increase pressure causing retinal hemorrhages and the classic "blood and thunder" appearance on funduscopic examination. In addition to these findings, the retinal veins may appear dilated and tortuous with macular and optic disc edema (unilateral). Ultimately the pressure increase leads to retinal ischemia and subsequent vision loss. The degree of vision loss depends on how much venous obstruction actually occurs. The cherry red fovea (A) is seen in cases of central retinal artery occlusion. Central retinal artery occlusion essentially causes an ischemic infarct of the retina. In this case, examination reveals a pale retina (B) with the fovea standing out as a cherry-red spot. Papilledema (C) is usually a bilateral physical examination finding where the optic disc is edematous with blurred edges. This is representative of conditions that cause increased intracranial pressure.

13-year-old girl presents with swelling on her eyelid as shown above. Which of the following is the most appropriate treatment? Eyelid scrub Incision and drainage Topical antibiotics Warm compresses

Correct Answer ( D ) Explanation: Hordeola are inflammations of the glands of Zeis or hair follicles. They typically begin as a more diffuse swelling of the lid that eventually coalesces into a nodule. There is often infection with Staphylococcus aureus contributing to the development of the hordeolum. However, treatment is with warm compresses applied 15 minutes four to six times a day. It may take weeks for these to resolve. Eyelid scrub (A) is the treatment for blepharitis, which is a chronic inflammation of the eyelid near the root of the lashes. Patients have itchy eyes with a foreign body sensation and on examination have red, swollen eyelids often with crusty and flaky skin. Topical antibiotics (C) are not recommended for the routine treatment of hordeola. They may be added if the hordeolum leads to significant associated blepharitis. Incision and drainage (B) is indicated for persistent hordeola that are refractory to conventional treatment and not typically considered until several weeks of failed therapy.

Which of the following is most characteristic of acute iritis? Elevated intraocular pressure Mid-dilated pupil that is sluggishly reactive Pain with extraocular muscle movement Photophobia

Correct Answer ( D ) Explanation: Iritis (aka anterior uveitis) is an inflammation of the anterior portion of the uvea (iris, ciliary body, and choroid). It is most commonly caused by trauma; the hallmark symptom is photophobia. Other signs and symptoms include decreased visual acuity, ciliary flush, and a deep ache that is not relieved by topical anesthetic drops. The slit lamp exam classically reveals a cell and flare appearance.Cell is the individual inflammatory cells, and flare is the foggy appearance given by protein that has leaked from inflamed blood vessels. Elevated intraocular pressure (A) and a mid-dilated pupil that is sluggishly reactive (B) are associated with acute closed-angle glaucoma. Iritis is associated with decreased intraocular pressure due to compromised aqueous production. The pupil in iritis is commonly miotic. Pain with extraocular muscle movement (C) is associated with optic neuritis and orbital cellulitis.

56-year-old man presents to the emergency department with acute onset right eye pain and vision loss. His past medical history is significant for hypertension, COPD and sickle cell disease. He states he first noticed the pain as he was turning the lights off in his room before going to sleep tonight. On physical exam there is conjunctival injection of the right eye. Vital signs are BP 130/85, HR 82, RR 12, temperature 98.2°F. Intraocular pressure of right eye measures 30 mm Hg, left eye 15 mm Hg. What treatment should be initiated in this patient? Acetazolamide intravenously Prednisone orally Tetracaine optic drops Timolol optic drops

Correct Answer ( D ) Explanation: Patients with acute angle closure glaucoma will often present with acute unilateral eye pain and visual loss. Patients can report seeing halos around lights as well. Physical exam in these patients reveals a "steamy" cornea and an elevated intraocular pressure. These symptoms develop secondary to increased intraocular pressure from a buildup of aqueous humor within the anterior chamber of the eye. Patients typically first notice the eye pain when they go from a bright to a dark room due to pupillary dilation. Treatment of acute angle closure glaucoma involves decreasing the intraocular pressure through a topical beta blocker such as timolol optic drops or intravenous acetazolamide. Patients can additionally benefit from being given pilocarpine optic drops, which is a miotic drug that prevents pupillary dilation Acetazolamide intravenously (A) should be avoided in this case due to the risk of causing increased sickling of RBCs in sickle cell patients. This is thought to be due to decreased pH in the anterior chamber of the eye caused by acetazolamide. Tetracaine optic drops (C) are generally only helpful in corneal abrasions. Topical steroids not oral prednisone (B) are more useful in this case because treatment can be directed locally to the affected eye.

Which of the following is most characteristic of acute conjunctivitis? Ciliary injection Diminished visual acuity Pain and photophobia Serous or clear discharge

Correct Answer ( D ) Explanation: The most common cause of a red eye is acute conjunctivitis, most commonly due to viral infection. Patients often complain of an itchy, irritated eye with serous or clear discharge. Purulent (creamy white or yellow watery) discharge suggests a bacterial cause. Scanty, white, stringy exudate occurs most often with allergic conjunctivitis. It is important to examine both eyes because many patients with conjunctivitis in one eye have signs of early conjunctivitis in the other. Conjunctival injection is characterized by individually visible vessels in the conjunctiva branching from the sclera toward the cornea. Inspect the palpebral conjunctiva carefully with magnification to determine whether lymphoid hyperplasia (cobblestone appearance) exists. The type and quantity of discharge are assessed by pulling down the lower lid. The absence of discharge should prompt investigation to other causes of a red eye. Ciliary injection (A) appears as engorgement of the deep vessels around the limbus. The significance of ciliary injection is that the deep ciliary vessels are involved, indicating a much more serious inflammatory condition of the eye. Diminished visual acuity (B) is not associated with acute conjunctivitis. If present, other serious causes of red eye should be investigated. Pain and photophobia (C) is associated with glaucoma and keratitis.

Which of the following is the most common etiology of external otitis? Aspergillus niger Candida Moraxella catarrhalis Pseudomonas aeruginosa

Correct Answer ( D ) Explanation: The most common cause of otitis externa is infection due to Pseudomonas aeruginosa and Staphylococcus aureus. The pain from otitis externa is caused by inflammation and edema of the ear canal skin, which is normally adherent to the bone and cartilage of the auditory canal. The inflammatory reaction can be caused by bacteria, fungi, or contact dermatitis. Cerumen protects the canal by forming an acidic coat that helps prevent infection. Factors that predispose to otitis externa include absence of cerumen, often from excessive cleaning by the patient, water that macerates the skin of the auditory canal and raises the pH, and trauma to the skin of the auditory canal from foreign bodies or use of cotton swabs. Treatment includes suction and gentle warm irrigation of the canal. 2% acetic acid solution or an alternative drying medication can be administered. A topical antibiotic drop with steroid is first-line therapy (neomycin/polymyxin/hydrocortisone). Use the suspension rather than the solution if the tympanic membrane is ruptured. Fungal infections compose less than 10% of external otitis cases. The most common fungi are Aspergillus niger (A) and Candida (B) species and they are more prevalent in tropical climates. Moraxella catarrhalis (C) is a common contributor to otitis media but is not often implicated in otitis externa.

31-year-old woman with diabetes mellitus type II presents with worsening left ear pain and fever for one week. Examination reveals an ill-appearing woman with a minimally swollen external auditory canal and swelling, warmth and tenderness around the left ear. The patient was started on antibiotic drops for her ear one week ago but her symptoms have worsened. What management is indicated? Continue topical antibiotics Incision and drainage Start high-dose amoxicillin Start intravenous ciprofloxacin and admit

Correct Answer ( D ) Explanation: The patient presents with necrotizing otitis externa (OE) and should have intravenous antibiotics that cover pseudomonal species started and admitted for further evaluation. Necrotizing or malignant OE is an aggressive infection that begins in the external auditory canal and progresses through the periauricular tissue and cartilaginous external auditory meatus. It spreads into the fissures of Santorini (clefts in the floor of the meatus) and can continue to spread to the temporal bone causing osteomyelitis of the base of the skull. It typically affects elderly patients and those with immunocompromise (diabetics, AIDS patients etc.). The most common bacterial pathogens are Pseudomonas, Staphylococcus aureus, Staphylococcus epidermidis, Proteus mirabilis and Klebsiella. When the disease extends deeper, it may cause cranial nerve palsies. Patients typically present with otorrhea, otalgia, headache and periauricular swelling. Often, they have been treated with a topical antibiotic for OE without improvement. Management requires antibiotic coverage with double pseudomonal coverage or ciprofloxacin, admission and consideration for surgical debridement. Topical antibiotics (A) do not penetrate deeply enough into the surrounding tissue to act against necrotizing OE. High-dose amoxicillin (C) is the treatment of choice for otitis media but is unlikely to be effective in necrotizing OE as it has minimal pseuodomonal activity. Additionally, oral antibiotics are unlikely to be effective if the patient is very ill. Although deep abscesses can be formed, incision and drainage (B) should be performed by an ENT/surgeon.

93-year-old man on aspirin presents to the ED with epistaxis. On physical exam, you note bleeding from bilateral nares and down the posterior pharynx. You are unable to visualize the source of bleeding. Which of the following vessels is most likely the source of bleeding in this patient? AFacial artery BKiesselbach plexus CLabial artery DSphenopalatine artery

Correct Answer ( D ) Explanation: This patient is exhibiting signs and symptoms of posterior epistaxis. Posterior epistaxis is less common than anterior epistaxis and is most commonly due to bleeding from the sphenopalatine artery, located at the posterior aspect of the middle nasal turbinate. Patients with posterior epistaxis typically complain of bleeding from both nostrils. Inspection of the posterior pharynx may reveal profuse bleeding. In treating epistaxis, start by having the patient gently blow his nose or suction out the blood. If the bleeding is profuse, apply cotton balls soaked in a topical anesthetic and vasoconstrictor for at least five minutes. A good option is 1% tetracaine plus 0.05% oxymetazoline solution. In posterior epistaxis, this may not achieve hemostasis or allow visualization of the location of bleeding. Management of posterior epistaxis should be with either a Foley catheter or dual balloon pack. A 10 to 14 French Foley catheter with a 30 cc inflatable balloon may be inserted past the site of the bleeding and inflated with 5 to 7 cc of air or saline. It should then be pulled back onto the site of the posterior bleed and inflated until it is snug. An anterior nasal pack should then be placed in both nares. A dual balloon pack is placed by anesthetizing the nare and advancing the pack past the site of the bleeding. The posterior balloon is inflated with 5 to 7 cc of saline or air and pulled back onto the site of bleeding. It is then further inflated until it is snug. The anterior balloon is then inflated. The opposite nare should be packed as well. Complications of posterior epistaxis packing include aspiration, hypoxia, hypercarbia, and symptomatic bradycardia. Prophylactic antibiotic use is controversial as there is no evidence to support a reduction in toxic shock syndrome with use. All patients with posterior packing should be admitted to a telemetry bed for further monitoring while the packing is in place.Correct Answer ( D ) Explanation: This patient is exhibiting signs and symptoms of posterior epistaxis. Posterior epistaxis is less common than anterior epistaxis and is most commonly due to bleeding from the sphenopalatine artery, located at the posterior aspect of the middle nasal turbinate. Patients with posterior epistaxis typically complain of bleeding from both nostrils. Inspection of the posterior pharynx may reveal profuse bleeding. In treating epistaxis, start by having the patient gently blow his nose or suction out the blood. If the bleeding is profuse, apply cotton balls soaked in a topical anesthetic and vasoconstrictor for at least five minutes. A good option is 1% tetracaine plus 0.05% oxymetazoline solution. In posterior epistaxis, this may not achieve hemostasis or allow visualization of the location of bleeding. Management of posterior epistaxis should be with either a Foley catheter or dual balloon pack. A 10 to 14 French Foley catheter with a 30 cc inflatable balloon may be inserted past the site of the bleeding and inflated with 5 to 7 cc of air or saline. It should then be pulled back onto the site of the posterior bleed and inflated until it is snug. An anterior nasal pack should then be placed in both nares. A dual balloon pack is placed by anesthetizing the nare and advancing the pack past the site of the bleeding. The posterior balloon is inflated with 5 to 7 cc of saline or air and pulled back onto the site of bleeding. It is then further inflated until it is snug. The anterior balloon is then inflated. The opposite nare should be packed as well. Complications of posterior epistaxis packing include aspiration, hypoxia, hypercarbia, and symptomatic bradycardia. Prophylactic antibiotic use is controversial as there is no evidence to support a reduction in toxic shock syndrome with use. All patients with posterior packing should be admitted to a telemetry bed for further monitoring while the packing is in place.

32-year-old woman presents with right eye pain and a foreign body sensation. A fluorescein enhancement is shown above. Which of the following is most likely indicated? Acyclovir and follow up Eye patching and follow up Lid eversion, oral antibiotics and follow up Lid eversion, topical antibiotics and follow up

Correct Answer ( D ) Explanation: This patient presents with a traumatic corneal abrasion requiring topical antibiotics and follow up. It is also important to perform lid eversion on all patients with corneal abrasion to ensure that the foreign body that caused the abrasion is no longer present. Corneal abrasions are common and cause pain, photophobia, deceased visual acuity and a foreign body sensation. A hallmark of corneal abrasion is the complete or near-complete relief of pain with topical anesthetic drops. Physical examination often reveals conjunctival injection and may show a gross abrasion. Addition of fluorescein and viewing with a cobalt blue light can enhance the abrasion. If the abrasion lies over the visual axis, acuity can be significantly reduced. Treatment involves careful inspection for any retained foreign body including under the eyelid. Antibiotic ointment or drops should be employed and follow up should be scheduled with an ophthalmologist in 24-48 hours. Patients who wear contacts require special consideration. Antibiotic drops in these patients should have pseudomonal coverage and contacts should not be worn until cleared by an ophthalmologist. Acyclovir (A) is required for the treatment of herpes keratitis, which forms a branching pattern defect on the cornea. Eye patching (B) should be avoided. Oral antibiotics (C) are not necessary for simple corneal abrasions.

A 55-year-old man presents with sudden onset of decreased vision and pain in his right eye, He reports vomiting twice prior to presentation. Examination reveals normal extraocular motions and a mid position pupil that does not react to light. What management should be pursued? Acetazolamide drops Metoprolol intravenous Tetracaine drops Timolol drops

Correct Answer ( D ) Explanation: This patient presents with signs and symptoms concerning for acute angle closure glaucoma and should be immediately treated with a topical beta blocker like timolol. Glaucoma represents an ocular neuropathy caused by increased intraocular pressure. Glaucoma occurs either as the result of increased aqueous humor production (open-angle) or due to decreased drainage of the aqueous humor (closed-angle). Open-angle glaucoma is slower in onset and progressively causes changes in vision. Primary angle closure glaucoma, on the other hand, occurs suddenly and can lead to rapid vision loss. It is typically precipitated by pupillary dilation (i.e. going into a dimly lit or dark room). Symptoms of angle closure glaucoma include eye pain, headache, nausea, vomiting and change in vision. The cornea may appear cloudy and the pupil midposition and dilated. The diagnosis can be definitively made by finding an increased intraocular pressure on tonometry. Treatment should be initiated promptly with topical timolol and pilocarpine (a miotic agent). In severe cases, intravenous acetazolamide should be added. Acetazolamide (A) is effective if given intravenously in severe cases. Intravenous beta blockers (B) do not play a role in management. Tetracaine (C) is a topical anesthetic and does not play a role in glaucoma management.

35-year-old man presents to the emergency department at 3:00 am complaining of intense bilateral eye pain, redness, and tearing that woke him from sleep fifteen minutes prior to arrival. Eye examination with fluorescein staining of the corneas reveals diffuse punctate corneal lesions as shown above. Further history would likely reveal which of the following? AAllergies to dust mites, tree pollen, pet dander, and cockroaches BContinuous contact lens usage for the past four months CCorrosive chemical exposure DWelding without eye protection earlier in the day

Correct Answer ( D ) Explanation: Ultraviolet keratitis is the result of corneal epithelial damage from exposure to intense UV light. It can be from welding without eye protection, prolonged exposure to sunlight, or using a tanning bed without eye protection. Signs and symptoms include intense eye pain, photophobia, redness, and tearing with diffuse punctate corneal lesions seen with fluorescein staining. Onset of symptoms is typically delayed for several hours. Treatment consists of topical cycloplegics, broad spectrum antibiotic eye ointment, and ophthalmologic follow-up. Allergies to dust mites, tree pollen, pet dander, and cockroaches (A) is a feature of allergic conjunctivitis. Allergic conjunctivitis is characterized by bilateral eye itching and conjunctival injection with a normal fluorescein examination. Continuous contact lens usage for the past four months (B) is a cause of corneal ulcers. Corneal ulcers are epithelial defects typically caused by Pseudomonas aeruginosa in the setting of continuous contact lens usage. They present with pain, redness, photophobia, and tearing, but are usually unilateral and consist of a single corneal lesion. Corrosive chemical exposure (C) can potentially cause diffuse corneal damage, but would not present with a delay in onset of symptoms.

Question: What algorithm is used to determine the likelihood of bacterial pharyngitis and the need for antibiotics?

Question: What algorithm is used to determine the likelihood of bacterial pharyngitis and the need for antibiotics?


Conjuntos de estudio relacionados

Chapter 10 - Innovative Strategies that Change the Nature of Competition

View Set

SIE Chapter 3: Understanding Trading, Customer Accounts, and Prohibited Activities

View Set

Compliance - Anti-Kickback Statute

View Set

OSCE helpful from HHA (head to toe assessment

View Set